Mathcenter Forum  

Go Back   Mathcenter Forum > คณิตศาสตร์โอลิมปิก และอุดมศึกษา > อสมการ
สมัครสมาชิก คู่มือการใช้ รายชื่อสมาชิก ปฏิทิน ข้อความวันนี้

ตั้งหัวข้อใหม่ Reply
 
เครื่องมือของหัวข้อ ค้นหาในหัวข้อนี้
  #46  
Old 24 ธันวาคม 2005, 11:48
warut warut ไม่อยู่ในระบบ
กระบี่ไร้สภาพ
 
วันที่สมัครสมาชิก: 24 พฤศจิกายน 2001
ข้อความ: 1,627
warut is on a distinguished road
Post

อ้างอิง:
ข้อความเดิมของคุณ devil jr.:
ขอแก้ข้างบนนะครับ

คูณด้วย (a-b)(a-c)(b-c)ครับ ไม่ใช่ (a-b)(b-c)(c-a)
น้อง devil jr. (และสมาชิกทุกท่าน) สามารถแก้ไขข้อความที่ตัวเองโพสต์ไปแล้วได้โดยคลิ้กที่รูปกระดาษ-ดินสอ ไม่จำเป็นต้องมาโพสต์ข้อความใหม่ครับ จริงๆผมส่งเป็นข้อความส่วนตัวไปบอกตั้งแต่ตอนที่น้อง devil jr. โพสต์ใหม่ๆแล้ว แต่คิดว่าคงไม่ได้เปิดอ่าน ก็เลยจำเป็นต้องโพสต์ออกอากาศครับ
ตอบพร้อมอ้างอิงข้อความนี้
  #47  
Old 31 ธันวาคม 2005, 20:21
Detective_Maths##3#5#2#3#6##'s Avatar
Detective_Maths##3#5#2#3#6## Detective_Maths##3#5#2#3#6## ไม่อยู่ในระบบ
สมาชิกใหม่
 
วันที่สมัครสมาชิก: 28 พฤศจิกายน 2005
ข้อความ: 1
Detective_Maths##3#5#2#3#6## is on a distinguished road
Icon18

^_^
1.(Bulgarian MC 1997) ให้ a,b,c เป็นจำนวนจริงบวก มีสมบัติว่า abc=1 จงพิสูจน์ว่า
$\frac{1}{1+a+b}+\frac{1}{1+b+c}+\frac{1}{1+c+a}\leq \frac{1}{2+a}+\frac{1}{2+b}+\frac{1}{2+c}$

2.ให้ a,b,c เป็นจำนวนจริงบวก โดยที่ $ab+bc+ca\leq 3abc$
จงพิสูจน์ว่า $a^3 + b^3 + c^3 \geq a+b+c $
*****
และอยากทราบว่า ทฤษฎีบทว่าด้วยค่ามัชฌิม คืออะไร (รุสึกว่าจะอยู่ในวิชา Real Analysis)
[[คือ... สงสัยมากตอนอ่านการพิสูจน์ A.M-G.M. ถ่วงน้ำหนัก ในหนังสืออสมการ ของ สอวน.]]
ใครรู้ก็ช่วยตอบด้วยนะคร้าบบ.

(ปล. แก้ Latex ให้นะครับ หัดลองเขียนได้จากห้องข้างบนครับ อย่าพยายามเอา Latex มาปนกับ UBBcode : , อ้อ เราสามารถแก้ความตัวเอง กับ ดูผลการโพสต์ได้นะครับ. : gon)
__________________
Mathematics is the queen of science, and algebras is the queen of mathematics.
#####Detective_Maths####
ตอบพร้อมอ้างอิงข้อความนี้
  #48  
Old 05 มกราคม 2006, 15:26
gools's Avatar
gools gools ไม่อยู่ในระบบ
บัณฑิตฟ้า
 
วันที่สมัครสมาชิก: 26 เมษายน 2004
ข้อความ: 390
gools is on a distinguished road
Post

ข้อ 1 ยังทำไม่ได้เลยครับ เหลือแค่วิธีเดียวแล้วคือถึกเอาแต่ยังไม่ได้ลอง

ส่วนข้อ 2 ครับ จากเงื่อนไขและ AM-HM จะได้ว่า
\[3 \geq \frac{1}{x}+\frac{1}{y}+\frac{1}{z} \geq \frac{9}{x+y+z}\]
ดังนั้น
\[x+y+z \geq 3\]
โดย Power Mean Inequality จะได้ว่า
\[x^3+y^3+z^3 \geq \frac{(x+y+z)^3}{9} = \frac{(x+y+z)(x+y+z)^2}{9} \geq x+y+z\]
ตอบพร้อมอ้างอิงข้อความนี้
  #49  
Old 05 มกราคม 2006, 23:22
gools's Avatar
gools gools ไม่อยู่ในระบบ
บัณฑิตฟ้า
 
วันที่สมัครสมาชิก: 26 เมษายน 2004
ข้อความ: 390
gools is on a distinguished road
Post

อ้างอิง:
ข้อความเดิมของคุณ Char Aznable:
ให้ x , y , z เป็นจำนวนจริงบวกที่มีผลบวกเป็น 3
จงพิสูจน์ว่า \[ \sqrt{x}+\sqrt{y}+\sqrt{z} \geq xy + yz + zx \]
แทน $x=a^2,y=b^2,z=c^2$ ดังนั้นคำถามใหม่คือ
ให้ $a , b , c$ เป็นจำนวนจริงบวกที่ $a^2+b^2+c^2=3$
จงพิสูจน์ว่า $a+b+c \geq a^2b^2+b^2c^2+c^2a^2$

เปลี่ยนรูปใหม่
\[\begin{array}{rcl}a+b+c &\geq& a^2b^2+b^2c^2+c^2a^2 \\
&=&\frac{(a^2+b^2+c^2)^2-(a^4+b^4+c^4)}{2} \\
&=& \frac{9-(a^4+b^4+c^4)}{2} \\
2(a+b+c)+(a^4+b^4+c^4) &\geq& 9 \\
\end{array}
\]

โดยอสมการ AM-GM จะได้ว่า
\[a^4+2a=a^4+a+a \geq 3\sqrt[3]{a^6}=3a^2\]

ดังนั้น
\[2(a+b+c)+(a^4+b^4+c^4) \geq 3(a^2+b^2+c^2) = 9\]

พิมพ์ง่ายขึ้นเยอะเลยครับ ขอบคุณมากครับ
ตอบพร้อมอ้างอิงข้อความนี้
  #50  
Old 07 มกราคม 2006, 14:06
Punk Punk ไม่อยู่ในระบบ
ลมปราณบริสุทธิ์
 
วันที่สมัครสมาชิก: 10 เมษายน 2005
ข้อความ: 108
Punk is on a distinguished road
Post

เพิ่มสักสามข้อละกันครับ ข้อแรกมาจากโจทย์ของน้อง Char Aznable ข้อสองขุดมาจากคลังข้อสอบเก่า
ส่วนข้อสามเป็นโจทย์ที่ไม่ได้ใช้ในการแข่ง IMO

1. กำหนดให้ $0\leq x,y\leq1$ จงพิสูจน์ว่า
$$
\frac{x}{1+x^2+y^2-x}+\frac{y}{1+x^2+y^2-y}+\frac{xy}{1+x^2+y^2-xy}\leq\frac{3}{2}
$$

2. นิยามลำดับ $\{a_n\}$ โดย
$$
a_n=1+\frac{1}{2+\frac{1}{3+\frac{\vdots}{n+\vphantom{\vdots}\frac{1}{\vphantom{\vdots}n+1}}}}
$$
จงพิสูจน์ว่า $|a_{100}-a_{99}|\leq1/(99!)^2$

3. จงหาจำนวนผลเฉลยที่เป็นจำนวนจริงบวกของสมการ
$$
a^{a^{a^x}}+a^{a^x}+a^x=3x
$$
เมื่อ $a=1/3$

07 มกราคม 2006 14:29 : ข้อความนี้ถูกแก้ไขแล้ว 1 ครั้ง, ครั้งล่าสุดโดยคุณ Punk
ตอบพร้อมอ้างอิงข้อความนี้
  #51  
Old 07 ตุลาคม 2006, 22:15
nooonuii nooonuii ไม่อยู่ในระบบ
ผู้พิทักษ์กฎทั่วไป
 
วันที่สมัครสมาชิก: 25 พฤษภาคม 2001
ข้อความ: 6,408
nooonuii is on a distinguished road
Post

ปลุกกระทู้ครับ
__________________
site:mathcenter.net คำค้น
ตอบพร้อมอ้างอิงข้อความนี้
  #52  
Old 08 ตุลาคม 2006, 21:33
nooonuii nooonuii ไม่อยู่ในระบบ
ผู้พิทักษ์กฎทั่วไป
 
วันที่สมัครสมาชิก: 25 พฤษภาคม 2001
ข้อความ: 6,408
nooonuii is on a distinguished road
Post

อ้างอิง:
ข้อความเดิมของคุณ Detective_Maths##3#5#2#3#6##:

1.(Bulgarian MC 1997) ให้ a,b,c เป็นจำนวนจริงบวก มีสมบัติว่า abc=1 จงพิสูจน์ว่า
$\frac{1}{1+a+b}+\frac{1}{1+b+c}+\frac{1}{1+c+a}\leq \frac{1}{2+a}+\frac{1}{2+b}+\frac{1}{2+c}$

โจทย์ข้อนี้ดู(ร้าย)เดียงสามาก น่าจะคิดออกด้วยวิธีง่ายๆ แต่พอทำจริงๆถึกพระเจ้าช่วยเลยครับ
อันนี้เป็นเฉลยจากหนังสือครับ

ให้ $x=a+b+c,y=ab+bc+ca$ จะได้ว่า $x,y\geq 3$ ดังนั้นอสมการสมมูลกับ
$$\displaystyle{ \frac{3+4x+y+x^2}{2x+y+x^2+xy}\leq\frac{12+4x+y}{9+4x+2y} }$$
จัดรูปใหม่ได้เป็น
$$(3x^2y-5x^2-12x)+(xy^2-y^2-3x-3y)+(6xy-9x-27)\geq 0$$
ซึ่งเป็นจริงเนื่องจากแต่ละวงเล็บมีค่ามากกว่าหรือเท่ากับ 0
__________________
site:mathcenter.net คำค้น
ตอบพร้อมอ้างอิงข้อความนี้
  #53  
Old 25 ตุลาคม 2006, 10:51
nooonuii nooonuii ไม่อยู่ในระบบ
ผู้พิทักษ์กฎทั่วไป
 
วันที่สมัครสมาชิก: 25 พฤษภาคม 2001
ข้อความ: 6,408
nooonuii is on a distinguished road
Post

เห็นว่ากระทู้ดูเหงาๆไม่มีคนแวะมาเยี่ยมเลย ขอเอาโจทย์ที่คิดเองมาลงไว้หน่อยละกันครับ ไม่ยากครับ

23. (ขอมั่วตัวเลขเอาละกันครับ ) ให้ $a,b,c > 0$ และ $abc=1$ จงพิสูจน์ว่า $$a^4+b^4+c^4\geq a^3+b^3+c^3$$
__________________
site:mathcenter.net คำค้น
ตอบพร้อมอ้างอิงข้อความนี้
  #54  
Old 25 ตุลาคม 2006, 11:40
SOS_math's Avatar
SOS_math SOS_math ไม่อยู่ในระบบ
จอมยุทธ์หน้าใหม่
 
วันที่สมัครสมาชิก: 10 กันยายน 2003
ข้อความ: 70
SOS_math is on a distinguished road
Post

ขอลองเล่นบ้าง จาก $abc=1$ จะได้ว่า $a^2+b^2+c^2\ge3$ เพราะว่า $\frac{a^2+b^2+c^2}{3}\ge \sqrt[3]{a^2b^2c^2}=1$
จากนั้นเราทราบว่า
$$\left(\frac{a^2+b^2+c^2}{3}\right)^{\frac12}\le\left(\frac{a^3+b^3+c^3}{3}\right)^{\frac13}$$
จึงได้ว่า
$$(a^2+b^2+c^2)^3\le 3(a^3+b^3+c^3)^2\le (a^2+b^2+c^2)(a^3+b^3+c^3)^2$$
หรือ $a^2+b^2+c^2\le a^3+b^3+c^3$
ต่อไปสุดท้ายแล้วโดย Cauchy-Schwarz-Bunyakovski Inequality
$$(a^3+b^3+c^3)^2\le (a^2+b^2+c^2)(a^4+b^4+c^4)\le (a^3+b^3+c^3)(a^4+b^4+c^4) $$
นั่นคือ $a^3+b^3+c^3 \le a^4+b^4+c^4$
ตอบพร้อมอ้างอิงข้อความนี้
  #55  
Old 26 ตุลาคม 2006, 03:51
nooonuii nooonuii ไม่อยู่ในระบบ
ผู้พิทักษ์กฎทั่วไป
 
วันที่สมัครสมาชิก: 25 พฤษภาคม 2001
ข้อความ: 6,408
nooonuii is on a distinguished road
Post

เยี่ยมครับคุณ SOS_math ผมมีอีกสองวิธีครับ

Fisrt Solution :

$\displaystyle{ (a^4-a^3)+(b^4-b^3)+(c^4-c^3) = (a-1)(a^3-1)+(b-1)(b^3-1)+(c-1)(c^3-1)+(a+b+c-3) }$
$=(a-1)^2(a^2+a+1)+(b-1)^2(b^2+b+1)+(c-1)^2(c^2+c+1)+(a+b+c-3)\geq 0$
เนื่องจาก $a+b+c\geq 3$ โดย AM-GM

Second Solution : โดย Power Mean inequality เราได้ว่า
$$\displaystyle{ \Bigg( \frac{a^4+b^4+c^4}{3} \Bigg) ^ {\frac{1}{4}} \geq \Bigg( \frac{a^3+b^3+c^3}{3} \Bigg) ^ {\frac{1}{3}} }$$
ดังนั้น
$$\displaystyle{ \frac{a^4+b^4+c^4}{3} \geq \Bigg( \frac{a^3+b^3+c^3}{3} \Bigg) ^ {\frac{4}{3}} \geq \frac{a^3+b^3+c^3}{3} }$$
เนื่องจาก $\frac{a^3+b^3+c^3}{3}\geq 1$ โดย AM-GM และ $x\geq 1 , p\geq 1 \Rightarrow x^p\geq x$
__________________
site:mathcenter.net คำค้น
ตอบพร้อมอ้างอิงข้อความนี้
  #56  
Old 26 ตุลาคม 2006, 04:00
nooonuii nooonuii ไม่อยู่ในระบบ
ผู้พิทักษ์กฎทั่วไป
 
วันที่สมัครสมาชิก: 25 พฤษภาคม 2001
ข้อความ: 6,408
nooonuii is on a distinguished road
Post

24. จงพิสูจน์ว่า $4(a-b+1)^2 \geq 1 - 2a + 2b$ สำหรับทุกจำนวนจริง $a,b$
__________________
site:mathcenter.net คำค้น
ตอบพร้อมอ้างอิงข้อความนี้
  #57  
Old 26 ตุลาคม 2006, 05:39
Punk Punk ไม่อยู่ในระบบ
ลมปราณบริสุทธิ์
 
วันที่สมัครสมาชิก: 10 เมษายน 2005
ข้อความ: 108
Punk is on a distinguished road
Post

ช่วยแจม ข้อนี้จำไม่ได้ว่าได้แต่ใดมา

25. กำหนดให้ $k,m,n$ เป็นจำนวนเต็มบวกซึ่ง
\[
\max\{k,m,n\}\leq 1004\min\{k,m,n\}
\]
จงพิสูจน์ว่า
\[
m^{1/k}n^{1/m}k^{1/n}\leq m^{1/k}+n^{1/m}+k^{1/n}+2006
\]
ตอบพร้อมอ้างอิงข้อความนี้
  #58  
Old 04 มีนาคม 2007, 00:31
nooonuii nooonuii ไม่อยู่ในระบบ
ผู้พิทักษ์กฎทั่วไป
 
วันที่สมัครสมาชิก: 25 พฤษภาคม 2001
ข้อความ: 6,408
nooonuii is on a distinguished road
Post

ขอปลุกกระทู้อีกรอบ ชุดนี้ไม่ยากมากครับ

26. $x,y>0$
$$\frac{x}{\sqrt{y}}+\frac{y}{\sqrt{x}}\geq \sqrt{x}+\sqrt{y}$$

27. $a,b,c>0$
$$\frac{ab}{a+b}+\frac{bc}{b+c}+\frac{ca}{c+a}\leq \frac{a+b+c}{2}$$

28. $x,y,z>0$
$$\frac{x^2}{y^2}+\frac{y^2}{z^2}+\frac{z^2}{x^2}\geq \frac{x}{y}+\frac{y}{z}+\frac{z}{x} $$

29. $x,y,z>0$
$$ \frac{x^2}{y^2}+\frac{y^2}{z^2}+\frac{z^2}{x^2}\geq \frac{y}{x}+\frac{z}{y}+\frac{x}{z} $$

30. $x,y,z>0$
$$xy+yz+zx\geq\sqrt{3xyz(x+y+z)} $$
__________________
site:mathcenter.net คำค้น
ตอบพร้อมอ้างอิงข้อความนี้
  #59  
Old 04 มีนาคม 2007, 01:51
M@gpie's Avatar
M@gpie M@gpie ไม่อยู่ในระบบ
ลมปราณไร้สภาพ
 
วันที่สมัครสมาชิก: 09 ตุลาคม 2003
ข้อความ: 1,227
M@gpie is on a distinguished road
Post

หลังจากที่มีอสมการในหัวสมองเพิ่มขึ้นขอลองฝึกฝีมือบ้าง ผิดถูกยังไงก็บอกด้วยนะครับ
26. โดยไม่เสียนัยทั่วไปสมมติให้ $x>y>0$
จะได้ว่า \[ \begin{array}{ccl} (\sqrt{x} - \sqrt{y})^2 & \geq & 0\\
x - 2\sqrt{xy} + y &\geq & 0 \\
x -\sqrt{xy} + y &\geq & \sqrt{xy} \\
{\displaystyle \frac{(\sqrt{x}+\sqrt{y})}{\sqrt{xy}}(x -\sqrt{xy} + y)} &\geq & {\displaystyle \frac{(\sqrt{x}+\sqrt{y})}{\sqrt{xy}} \sqrt{xy}} \\
{\displaystyle \frac{\sqrt{x}^3 + \sqrt{y}^3}{\sqrt{xy}} }&\geq & \sqrt{x}+\sqrt{y} \\
{\displaystyle \frac{x}{\sqrt{y}} + \frac{y}{\sqrt{x}}} & \geq & \sqrt{x} +\sqrt{y}
\end{array}\]

27. โดย $H.M. \leq A.M.$ จะได้ว่า
\[ \frac{ab}{a+b} = \frac{1}{\frac{1}{a}+\frac{1}{b}} \leq \frac{a+b}{4}\]
ในทำนองเดียวกัน
\[ \frac{bc}{b+c} = \frac{1}{\frac{1}{b}+\frac{1}{c}} \leq \frac{b+c}{4}\]
\[ \frac{ac}{a+c} = \frac{1}{\frac{1}{c}+\frac{1}{a}} \leq \frac{c+a}{4}\]
นำสามอสมการบวกกันจะได้ อสมการที่ต้องการ

29. เปลี่ยนตัวแปรโดยให้ $a=\frac{x}{y}, \; \; b=\frac{y}{z}, \;\; c=\frac{z}{x} $
จะได้ว่าอสมการที่ต้องพิสูจน์กลายเป็น
\[ a^2+b^2+c^2 \geq \frac{1}{a} +\frac{1}{b} +\frac{1}{c}\]
โดยที่ $abc=1$
พิจารณา \[ (a-b)^2 +(b-c)^2 +(a-c)^2 \geq 0 \Rightarrow a^2+b^2+c^2 \geq ab+bc+ac = \]
โดยเงื่อนไข $abc=1$ จะได้ว่า \[ a^2+b^2+c^2 \geq \frac{1}{a} + \frac{1}{b} +\frac{1}{c}\] เมื่อแทนค่า $a,b,c$ กลับจะได้อสมการที่ต้องการ
ที่เหลือเดี๋ยวมาคิดต่อครับ
__________________
PaTa PatA pAtA Pon!

04 มีนาคม 2007 02:10 : ข้อความนี้ถูกแก้ไขแล้ว 1 ครั้ง, ครั้งล่าสุดโดยคุณ M@gpie
ตอบพร้อมอ้างอิงข้อความนี้
  #60  
Old 04 มีนาคม 2007, 03:58
nooonuii nooonuii ไม่อยู่ในระบบ
ผู้พิทักษ์กฎทั่วไป
 
วันที่สมัครสมาชิก: 25 พฤษภาคม 2001
ข้อความ: 6,408
nooonuii is on a distinguished road
Post

26. ใช้เงื่อนไข $x>y>0$ ตรงไหนครับ

29. ใช้อสมการโคชีก็ได้ครับ

$$\frac{1}{a}+\frac{1}{b}+\frac{1}{c} = ab+bc+ca\leq a^2+b^2+c^2$$
__________________
site:mathcenter.net คำค้น
ตอบพร้อมอ้างอิงข้อความนี้
ตั้งหัวข้อใหม่ Reply


หัวข้อคล้ายคลึงกัน
หัวข้อ ผู้ตั้งหัวข้อ ห้อง คำตอบ ข้อความล่าสุด
Algebra Marathon nooonuii พีชคณิต 199 20 กุมภาพันธ์ 2015 10:08
Trigonometric Marathon Mastermander พีชคณิต 251 24 พฤศจิกายน 2013 21:21
Calculus Marathon (2) nongtum Calculus and Analysis 134 03 ตุลาคม 2013 16:32
Marathon Mastermander ฟรีสไตล์ 6 02 มีนาคม 2011 23:19
Calculus Marathon nooonuii Calculus and Analysis 222 26 เมษายน 2008 03:52


กฎการส่งข้อความ
คุณ ไม่สามารถ ตั้งหัวข้อใหม่ได้
คุณ ไม่สามารถ ตอบหัวข้อได้
คุณ ไม่สามารถ แนบไฟล์และเอกสารได้
คุณ ไม่สามารถ แก้ไขข้อความของคุณเองได้

vB code is On
Smilies are On
[IMG] code is On
HTML code is Off
ทางลัดสู่ห้อง


เวลาที่แสดงทั้งหมด เป็นเวลาที่ประเทศไทย (GMT +7) ขณะนี้เป็นเวลา 06:40


Powered by vBulletin® Copyright ©2000 - 2024, Jelsoft Enterprises Ltd.
Modified by Jetsada Karnpracha